\documentclass{exam} \usepackage{amsmath} \usepackage{amssymb} %\usepackage{color} %\shadedsolutions% \printanswers \pointsinmargin \marginpointname{ \points} %\pointsinrightmargin %\bracketedpoints %\boxedpoints \addpoints \pagestyle{headandfoot} \extraheadheight{1in} \headrule \footrule \lhead{\large\bf MATH-165\\Calculus I} \chead{Quiz \#14\\} \rhead[\Large\bf Name:\enspace\hbox to 2in{\hrulefill}]{} \lfoot{Teaching Assistant: Bethany Kubik} \cfoot{Page \thepage\ of \numpages} \rfoot{} %\qformat{Question \thequestion\ \hrulefill\ \thepoints} \renewcommand{\solutiontitle}{\noindent\textbf{Solution:}\par\noindent} \begin{document} \begin{center} %insert quote here\\ \fbox{\fbox{\parbox{.9\linewidth}{\centering The majority of the credit you receive will be based on the completeness and the clarity of your responses. Show your work, and avoid saying things that are untrue, ambiguous, or nonsensical.\\This quiz has \numquestions\ questions, for a total of \numpoints\ points.\\}}} \end{center} \begin{questions} %%%%%%%%%%%%%%%%%%%%%%%%%%%%%%%%%%%%%%%%%%%%%%%%%%%%%%%%%%%%%%%%%%%%%%%%%%%%%%%%%%%%%%%%%%%%%%%% \question[5] Find $f(x)$ if $f'(x)=\cos(x) +x^3+7$ \vspace{2.4in} %%%%%%%%%%%%%%%%%%%%%%%%%%%%%%%%%%%%%%%%%%%%%%%%%%%%%%%%%%%%%%%%%%%%%%%%%%%%%%%%%%%%%%%%%%%%%%%% \question[5] Write the Riemann sum in integral notation then evaluate $$\lim_{n\rightarrow \infty}\sum_{i=1}^{n}(\frac{2}{n})\sqrt{4-(\frac{2}{n}i)^2})$$ \vspace{2.4in} %%%%%%%%%%%%%%%%%%%%%%%%%%%%%%%%%%%%%%%%%%%%%%%%%%%%%%%%%%%%%%%%%%%%%%%%%%%%%%%%%%%%%%%%%%%%%%%% \end{questions} \end{document} \bye